由买买提看人间百态

boards

本页内容为未名空间相应帖子的节选和存档,一周内的贴子最多显示50字,超过一周显示500字 访问原贴
Quant版 - 一道题
相关主题
问一道面试题 brownian motion的一个 interatedBM 问题
brownian motion, got an answer but do not feel confident. Hcal expected value
local martingale发几道今天的海选考试题
question(brownian motion)问个应该比较简单的概率问题
Lognormal Random Walk问个面世题
GBM的FPT很straightforward么?问一个Shreve V2上的问题
Double Barrier 怎么超级麻烦?[合集] A Brownian Motion Question
问两道pricing的题这道题, 我做得对马?(stochastic process)
相关话题的讨论汇总
话题: mu话题: 150话题: 60话题: stock话题: gbm
进入Quant版参与讨论
1 (共1页)
B*****9
发帖数: 48
1
suppose current stock price is $60. the stock price follows a geometric
brownian motion with parameter (mu, sigma)what's the expected time that the
stock price hits 150?
f*****s
发帖数: 141
2
ln(S_t/S_0) starts from zero, hit ln(150/160), use exponential martingale to
solve this.
h*y
发帖数: 1289
3
why ln(150/160), typo?

to

【在 f*****s 的大作中提到】
: ln(S_t/S_0) starts from zero, hit ln(150/160), use exponential martingale to
: solve this.

f*****s
发帖数: 141
4
Seems the exponential martingale approach does not work, since P(hit 150) =
1.
Can we directly use E[S_t] = 60 * exp{mu t} = 150, then solve t?
B*****9
发帖数: 48
5
should be 60 * exp{(mu-sigma^2/2)t}?
if mu
=
?

【在 f*****s 的大作中提到】
: Seems the exponential martingale approach does not work, since P(hit 150) =
: 1.
: Can we directly use E[S_t] = 60 * exp{mu t} = 150, then solve t?
:

f*****s
发帖数: 141
6
Sorry, this GBM, E(S_t)=60exp(mu*t)=150 ==>t = 1/mu * log(150/60)
Is this correct?
f*****s
发帖数: 141
7
Sorry, this GBM, E(S_t)=60exp(mu*t)=150 ==>t = 1/mu * log(150/60)
Is this correct?
c**********s
发帖数: 295
8
log(GBM) is a BM with drift, then use change of measure and reflection.
b**********5
发帖数: 51
9
I think we can use E[S_t] = 60 * exp{mu t} = 150, then solve t. See the
following linkage at Wiki:
http://en.wikipedia.org/wiki/Geometric_Brownian_motion
M*****y
发帖数: 666
10
not agree.
it is one of stochastic process problems involving ODE method
the result is little complicated
1 (共1页)
进入Quant版参与讨论
相关主题
这道题, 我做得对马?(stochastic process)Lognormal Random Walk
一个百思不得其解 的 Martingale stopping time 问题GBM的FPT很straightforward么?
一个关于lognormal的简单问题Double Barrier 怎么超级麻烦?
有没有熟悉change numeraire的?关于利率model,问两道pricing的题
问一道面试题 brownian motion的一个 interatedBM 问题
brownian motion, got an answer but do not feel confident. Hcal expected value
local martingale发几道今天的海选考试题
question(brownian motion)问个应该比较简单的概率问题
相关话题的讨论汇总
话题: mu话题: 150话题: 60话题: stock话题: gbm